0 Daumen
140 Aufrufe

Aufgabe:

Man bestimme k ∈ ℝ so, dass die Vektoren

U = (1, -1, 2)     und.  V=(k²,k,-3)

Orthogonal sind.



Problem/Ansatz:

Ist 3 und -2 richtig?

Bitte klären sie mich auf.

Avatar von

1 Antwort

0 Daumen

Richtig!

............

:-)

Avatar von 47 k

Ein anderes Problem?

Stell deine Frage

Willkommen bei der Mathelounge! Stell deine Frage einfach und kostenlos

x
Made by a lovely community